Search found 11 matches


is the answe A? let me explain why i think so.Firstly,i am assumimg that the two pools have equal capacity. A s sufficient because we can deduce using the stem that what is added to pool A is 4/7 q'ty of water(6/7 -2/7) which is the quantity in pool Y and you can proceed from there. B is not suffici...

by floravaze

Tue Aug 12, 2008 2:28 am
Forum: Data Sufficiency
Topic: Filling the pool
Replies: 6
Views: 2370

thanks guys,u really broke it down for me.

by floravaze

Thu Jul 31, 2008 8:53 am
Forum: Problem Solving
Topic: Plz help with explanatinos!!
Replies: 6
Views: 1665

Plz help with explanatinos!!

According to the directions on a can of frozen juice concentrate,1 can of concentrate is to be mixed with 3 cans of water to make orange juice.How many 12-ounce cans of the concentrate are required to prepare 200 6-ounce servings of orange juice? Answer is 25 All of the stocks on the over-the counte...

by floravaze

Thu Jul 31, 2008 8:08 am
Forum: Problem Solving
Topic: Plz help with explanatinos!!
Replies: 6
Views: 1665

Hi Ian;
do post the complete answer.. i am quite stuck.i know that the min value is -1 but how do i get the max value?please

by floravaze

Fri Jul 25, 2008 10:41 am
Forum: Problem Solving
Topic: Writing a number as sum of ....
Replies: 8
Views: 1582

Gee-i got it.Thank you so much.

by floravaze

Thu Jul 17, 2008 5:42 am
Forum: Problem Solving
Topic: digits fun!
Replies: 4
Views: 2437

Please can you explain your logic in the first step?i do not have a clue what it means.please!!!!

by floravaze

Thu Jul 17, 2008 5:16 am
Forum: Problem Solving
Topic: digits fun!
Replies: 4
Views: 2437

Thanks Beeparo
U're right about my example.it is undefinable and not zero.I really do make these stupid mistakes at times.

by floravaze

Sat Jun 28, 2008 2:35 am
Forum: Data Sufficiency
Topic: need help with method...
Replies: 4
Views: 1779

I think the 1st statement is insufficient because in order to know the percentage increase,we need to know original number as well as the increase,in which case,only the increase is known. Also,statement 2 is insufficient because it gives only overall percentage increase which could have been caused...

by floravaze

Fri Jun 27, 2008 6:10 am
Forum: Data Sufficiency
Topic: gmat prep
Replies: 1
Views: 1539

i seriously doubt that the answer is B.I'd rather go with E. Statement 1 is insufficient because it tells you nothing about y and statement 2 is also insufficient for the same reason that gives no infomation about x.Now,the two statements taken together are also insufficient coz the relative magnitu...

by floravaze

Fri Jun 27, 2008 4:28 am
Forum: Data Sufficiency
Topic: need help with method...
Replies: 4
Views: 1779

hi, first,are you sure the answer is A.Because it seems to me like the right answer should be E.let me explain. when M is divided by 6,remainder is 1,so let's have an equation M/6=x + 1 When N is divided by 6,remainder is 3,so N/6=y + 3 When we cross-multiply,we get M=6x +6 and N=6y+18 Therefore M+N...

by floravaze

Fri Jun 27, 2008 3:08 am
Forum: Problem Solving
Topic: any other method
Replies: 7
Views: 1582

Sorry,not all of us on this forum have access to this book.Well,let me speak for myself.i don't have access to the book,so could you kindly please post the question so we can share?

by floravaze

Thu Jun 26, 2008 5:28 am
Forum: Problem Solving
Topic: Princeton Review Math Bin 4 Question
Replies: 3
Views: 2030